Ernesto: Sales of VCRs—videocassette recorders—will decline

This topic has expert replies
User avatar
Master | Next Rank: 500 Posts
Posts: 127
Joined: Tue May 13, 2008 12:28 am
Thanked: 2 times
GMAT Score:710
Ernesto: Sales of VCRs—videocassette recorders—will decline in the next few years because the saturation level among U.S. households has virtually been reached.

Milton: Every year a greater number of popular feature films is released on cassette—at least 6 per month.Clearly VCR sales will remain constant, if not rise.

Which of the following is the best logical evaluation of Milton’s response to Ernesto’s argument?

(A) He cites evidence that, if true, disproves the evidence cited by Ernesto in drawing his conclusion.
(B) He points out a gap in the logic followed by Ernesto in drawing his conclusion.
(C) He cites an issue ignored by Ernesto and which outweighs the issues raised by Ernesto.
(D) He does not speak to Ernesto’s point because he fails to raise the issue of whether VCR sales may be linked to sales of other leisure-related products.
(E) He fails to respond to Ernesto’s argument because he assumes that nothing will significantly retard the sale of VCR’s,which was the issue that Ernesto raised.
Thanks
Airan

Junior | Next Rank: 30 Posts
Posts: 27
Joined: Mon Apr 28, 2008 2:18 pm
Thanked: 1 times

by Anon » Mon Jul 07, 2008 10:16 am
(E) He fails to respond to Ernesto’s argument because he assumes that nothing will significantly retard the sale of VCR’s,which was the issue that Ernesto raised.

User avatar
Junior | Next Rank: 30 Posts
Posts: 13
Joined: Wed May 18, 2011 10:43 am
Location: Saint-Petersburg, Russia
GMAT Score:560

by kiton » Sat Jun 11, 2011 3:27 am
Hello! Why not A? Please, explain.

E says that "Milton assumes that nothing will significantly retard...". Though Milton does not assume anything, he clearly cites evidence, which disproves Ernesto's evidence in drawing his conclusion.

Junior | Next Rank: 30 Posts
Posts: 22
Joined: Thu Feb 24, 2011 4:47 am
Thanked: 1 times

by sunilramu » Sat Jun 11, 2011 5:00 am
IMO E

Junior | Next Rank: 30 Posts
Posts: 22
Joined: Thu Feb 24, 2011 4:47 am
Thanked: 1 times

by sunilramu » Sat Jun 11, 2011 5:01 am
IMO E

User avatar
Junior | Next Rank: 30 Posts
Posts: 13
Joined: Wed May 18, 2011 10:43 am
Location: Saint-Petersburg, Russia
GMAT Score:560

by kiton » Sat Jun 11, 2011 5:15 am
Hmm... I think i am looking at logical structure (choosing A), not logical evaluation. What is the strategy here?

User avatar
Junior | Next Rank: 30 Posts
Posts: 13
Joined: Wed May 18, 2011 10:43 am
Location: Saint-Petersburg, Russia
GMAT Score:560

by kiton » Sat Jun 11, 2011 5:16 am
Hmm... I think i am looking at logical structure (choosing A), not logical evaluation. What is the strategy here?

Junior | Next Rank: 30 Posts
Posts: 14
Joined: Sun Jun 06, 2010 8:21 am

by sainagalla » Sat Jun 11, 2011 5:17 am
why not C?

Junior | Next Rank: 30 Posts
Posts: 26
Joined: Sat May 08, 2010 7:29 am

by badresh70 » Sat Jun 11, 2011 6:31 am
I ll go for option "A".

@Airan,

can you post the answer with explanation.

Regards,
Badresh

Senior | Next Rank: 100 Posts
Posts: 33
Joined: Wed Jan 11, 2012 5:18 pm

by nailGmat2012 » Tue Jan 24, 2012 6:32 pm
what's the strategy to solve this? I know that OA is E, but cannot really absorb it.

Thanks

Senior | Next Rank: 100 Posts
Posts: 62
Joined: Fri Aug 12, 2011 12:37 am
Thanked: 3 times
Followed by:1 members

by sk8legend408 » Wed Jan 25, 2012 10:21 am
Milton addresses Ernesto's argument by talking about something that does not directly respond to his point. Ernesto talks about VCR sales and Milton talks about more VCR films released this year which will make people buy more VCRs. It may make people buy more tapes, but not necessarily more VCRs. I already own a DVD player, I would only buy a DVD if a good movie came out...not the whole player.

A is wrong because Milton's evidence would not disprove Ernesto's evidence because even though more VCR tapes might be sold, more VCRs themselves may not follow the same pattern.

B is also wrong because he doesn't point to a gap.

C is also wrong because he does not cite an issue ignored by Ernesto.

D is wrong because nothing is mentioned about other leisure-related products.

You are left with the answer...E.